How to find upper bound for recurrence relation

Click For Summary
To find a tight upper bound for the recurrence relation T(n) = T(n/2) + T(n/3) + c, a recursion tree approach is suggested, though challenges arise due to the asymmetry of the tree. The lack of symmetry complicates the analysis, as one side of the tree can extend indefinitely without reaching a base case like T(1). It is noted that for the recurrence to be valid, n must be a multiple of 6, leading to the consideration of sequences such as T(6n) and T(36n). The discussion raises concerns about defining T(n) for indices that are not multiples of 12 or 18, highlighting the need for clarity in the recurrence's domain. Understanding these constraints is crucial for accurately solving the problem.
ciphone
Messages
3
Reaction score
0

Homework Statement


Find a tight upper bound for the recurrence relation using a recursion tree argument

Homework Equations


T(n)=T(n/2)+T(n/3)+c

The Attempt at a Solution


I don't know how to do this problem because the tree doesn't have symmetry. One side of the tree can keep going because of the lack of symmetry plus at the end there is no way that you get T(1). How do you solve this problem using a recursion tree?
 
Physics news on Phys.org
I don't understand.
If one consider that the indexation of sequence ##T(n)## is in ##\mathbb{N}-\{0\}##, then the index ##n## must be a multiple of ##6##, since 2 and 3 are mutually prime divisors of ##n##.

At first sight you must consider a sequence of type ##T(6n) = T(3n) + T(2n) + c ##. But for 6 to divide ##3n## and ##2n##, it must divide ##n##. It leads you to consider the sequence ##T(36n) = T(18n) + T(12n) + c ##.

With that, what sense do you give to ##T(n)## for indexes that are not multiple of 12 and 18 ?
 
Question: A clock's minute hand has length 4 and its hour hand has length 3. What is the distance between the tips at the moment when it is increasing most rapidly?(Putnam Exam Question) Answer: Making assumption that both the hands moves at constant angular velocities, the answer is ## \sqrt{7} .## But don't you think this assumption is somewhat doubtful and wrong?

Similar threads

  • · Replies 11 ·
Replies
11
Views
2K
  • · Replies 2 ·
Replies
2
Views
2K
  • · Replies 1 ·
Replies
1
Views
2K
  • · Replies 16 ·
Replies
16
Views
3K
  • · Replies 43 ·
2
Replies
43
Views
4K
Replies
8
Views
3K
  • · Replies 4 ·
Replies
4
Views
2K
  • · Replies 3 ·
Replies
3
Views
1K
Replies
13
Views
3K
  • · Replies 2 ·
Replies
2
Views
4K